Last visit was: 24 Apr 2024, 13:20 It is currently 24 Apr 2024, 13:20

Close
GMAT Club Daily Prep
Thank you for using the timer - this advanced tool can estimate your performance and suggest more practice questions. We have subscribed you to Daily Prep Questions via email.

Customized
for You

we will pick new questions that match your level based on your Timer History

Track
Your Progress

every week, we’ll send you an estimated GMAT score based on your performance

Practice
Pays

we will pick new questions that match your level based on your Timer History
Not interested in getting valuable practice questions and articles delivered to your email? No problem, unsubscribe here.
Close
Request Expert Reply
Confirm Cancel
SORT BY:
Date
Tags:
Difficulty: 605-655 Levelx   Assumptionx               
Show Tags
Hide Tags
User avatar
Manager
Manager
Joined: 10 Jan 2006
Posts: 64
Own Kudos [?]: 709 [210]
Given Kudos: 0
Send PM
Most Helpful Reply
Tutor
Joined: 16 Oct 2010
Posts: 14817
Own Kudos [?]: 64899 [39]
Given Kudos: 426
Location: Pune, India
Send PM
Target Test Prep Representative
Joined: 14 Oct 2015
Status:Founder & CEO
Affiliations: Target Test Prep
Posts: 18756
Own Kudos [?]: 22046 [20]
Given Kudos: 283
Location: United States (CA)
Send PM
General Discussion
User avatar
Manager
Manager
Joined: 19 Aug 2006
Posts: 112
Own Kudos [?]: 589 [1]
Given Kudos: 0
Send PM
Re: The program to control the entry of illegal drugs into the country was [#permalink]
1
Kudos
The program to control the entry of illegal drugs into the country was a failure in 1987. If the program had been successful, the wholesale price of most illegal drugs would not have dropped substantially in 1987.

Here author is considering the program failure is the only cause for drop in prices.
13. The argument in the passage depends on which of the following assumptions?

(A) The supply of illegal drugs dropped substantially in 1987.
(B) The price paid for most illegal drugs by the average consumer did not drop substantially in 1987.
(C) Domestic production of illegal drugs increased at a higher rate than did the entry of such drugs into the country.
(D) The wholesale price of a few illegal drugs increased substantially in 1987.
(E) A drop in demand for most illegal drugs in 1987 was not the sole cause of the drop in their wholesale price.
( If failure of the program is the only cause for drop in prices then it is surely preassumed that " A drop in demand for most illegal drugs in 1987 was not the sole cause of the drop in their wholesale price." )
User avatar
BSchool Moderator
Joined: 28 May 2012
Posts: 83
Own Kudos [?]: 416 [1]
Given Kudos: 11
Location: India
Concentration: General Management, Strategy
GPA: 3.33
WE:Information Technology (Retail)
Send PM
Re: The program to control the entry of illegal drugs into the country was [#permalink]
1
Kudos
carcass wrote:
The program to control the entry of illegal drugs into the country was a failure in 1987. If the program had been successful, the wholesale price of most illegal drugs would not have dropped substantially in 1987.

The argument in the passage depends on which of the following assumptions?


1) The supply of illegal drugs dropped substantially in 1987.

2)The price paid for most illegal drugs by the average consumer did not drop substantially in 1987.

3)Domestic production of illegal drugs increased at a higher rate than did the entry of such drugs into the country.

4)The wholesale price of a few illegal drugs increased substantially in 1987.

5)A drop in demand for most illegal drugs in 1987 was not the sole cause of the drop in their wholesale price.

Guys try this one. OA later . Thanks for discussion in advance :)


Let me just try -

1) The program to control the entry of illegal drugs into the country was a failure in 1987 - The program was a failure

2) If the program had been successful, the wholesale price of most illegal drugs would not have dropped substantially in 1987.

Thus, it means that the program was not successful ( there was illegal entry of drugs into the country) and hence, the prices dropped.

If I negate E - A drop in demand for most illegal drugs in 1987 was THE sole cause of the drop in their wholesale price.

okay - so if the price decrease was due to the drop in demand; the illegal entry of drugs was not the reason of their wholesale price decrease.

Am I right ?
Alum
Joined: 19 Mar 2012
Posts: 4341
Own Kudos [?]: 51447 [2]
Given Kudos: 2326
Location: United States (WA)
Concentration: Leadership, General Management
Schools: Ross '20 (M)
GMAT 1: 760 Q50 V42
GMAT 2: 740 Q49 V42 (Online)
GMAT 3: 760 Q50 V42 (Online)
GPA: 3.8
WE:Marketing (Non-Profit and Government)
Send PM
Re: The program to control the entry of illegal drugs into the country was [#permalink]
2
Kudos
Expert Reply
soniedarshan wrote:
The program to control the entry of illegal drugs into the country was a failure in
1987. If the program had been successful, the wholesale price of most illegal drugs
would not have dropped substantially in 1987.

The argument in the passage depends on which of the following
assumptions?

(A) The supply of illegal drugs dropped substantially in 1987.
(B) The price paid for most illegal drugs by the average consumer did not drop
substantially in 1987.
(C) Domestic production of illegal drugs increased at a higher rate than did the entry
of such drugs into the country.
(D) The wholesale price of a few illegal drugs increased substantially in 1987.
(E) A drop in demand for most illegal drugs in 1987 was not the sole cause of the drop
in their wholesale price.


E is correct. If you negate E, which becomes "Drop in demand WAS the sole cause of drop of wholesale price", the argument explodes into smithereens.
Intern
Intern
Joined: 09 Jan 2018
Posts: 12
Own Kudos [?]: 19 [2]
Given Kudos: 1059
Send PM
Re: The program to control the entry of illegal drugs into the country was [#permalink]
2
Kudos
MBA2ran wrote:
The program to control the entry of illegal drugs into the country was a failure in 1987. If the program had been successful, the wholesale price of most illegal drugs would not have dropped substantially in 1987.

The argument in the passage depends on which of the following assumptions?

(A) The supply of illegal drugs dropped substantially in 1987.
(B) The price paid for most illegal drugs by the average consumer did not drop substantially in 1987.
(C) Domestic production of illegal drugs increased at a higher rate than did the entry of such drugs into the country.
(D) The wholesale price of a few illegal drugs increased substantially in 1987.
(E) A drop in demand for most illegal drugs in 1987 was not the sole cause of the drop in their wholesale price.



The only choice that must be true in order to conclude legitimately from the drop in wholesale price of illegal drugs that the program was a failure is choice E, the best answer. If the drop in price was caused by a drop in demand, there is no reason to suspect that there has been any increase in supply caused by drugs entering the country.

The other choices can be false without affecting the argument. The supply of illegal drugs need not have dropped (choice A), and the retail price could have dropped (choice B). The entry of illegal drugs could have risen at a higher rate than domestic production (choice C), and no illegal drug need have undergone a substantial price rise (choice D).
Manager
Manager
Joined: 20 Aug 2017
Posts: 96
Own Kudos [?]: 200 [0]
Given Kudos: 174
Send PM
Re: The program to control the entry of illegal drugs into the country was [#permalink]
MBA2ran wrote:
The program to control the entry of illegal drugs into the country was a failure in 1987. If the program had been successful, the wholesale price of most illegal drugs would not have dropped substantially in 1987.


The conclusion in this argument is that the program to control the entry of illegal drugs into the country was a failure.
The argument used evidence that if the program had been successful, then the wholesale prices of illegal drugs would not have dropped substantially.
So, the argument assumes that change in the wholesale price of drugs is related to the success or failure of the program. there is no other reason for the drop in prices.

Quote:
(A) The supply of illegal drugs dropped substantially in 1987.

this statement tells us that only the supply dropped. no information about demand. if demand increased and supply dropped, then the prices should rise.
if demand also decreased then what. eliminated.

Quote:
(B) The price paid for most illegal drugs by the average consumer did not drop substantially in 1987.

if the price did not increase then it is related to sales. it means that the sales were constant. irrelevant. eliminated

Quote:
(C) Domestic production of illegal drugs increased at a higher rate than did the entry of such drugs into the country.

domestic production increased it means that it shows another cause that because of local production, there was a decrease in prices. but we need to make sure that there is no other cause. eliminated.

Quote:
(D) The wholesale price of a few illegal drugs increased substantially in 1987.

A FEW is a small number. if the wholesale price of a few increased that means the program would have been a success. but still if out of 1000 drugs, prices of 2 drugs increased then it does not lead to the conclusion. eliminated.

Quote:
(E) A drop in demand for most illegal drugs in 1987 was not the sole cause of the drop in their wholesale price.

this option tells us that the drop in demand was not the SOLE cause in the drop in prices. it means that there was some other cause. that cause was the failure of the program. correct.
Manager
Manager
Joined: 15 May 2017
Status:Discipline & Consistency always beats talent
Posts: 146
Own Kudos [?]: 124 [0]
Given Kudos: 132
Location: United States (CA)
GPA: 3.59
WE:Sales (Retail)
Send PM
The program to control the entry of illegal drugs into the country was [#permalink]
The program to control the entry of illegal drugs into the country was a failure in 1987. If the program had been successful, the wholesale price of most illegal drugs would not have dropped substantially in 1987.

Program to control the entry of illegal drugs -> failure
Program successful -> no drop in price
Drop in price -> Program unsuccessful

The reasoning is actually well connected. I don't find any missing link here. So this should be a defender assumption question type. We must defend that the failure of the program was the reason that caused price drop. Any answer choices that refute this will be wrong.

The argument in the passage depends on which of the following assumptions?


Quote:
(A) The supply of illegal drugs dropped substantially in 1987.

This, if anything, weakens the argument. It is saying that the program actually successful and that something else must have caused price drop. (A) is out.
Quote:
(B) The price paid for most illegal drugs by the average consumer did not drop substantially in 1987.

What the price actually was is irrelevant. What moved the price was the central point. (B) is out.
Quote:
(C) Domestic production of illegal drugs increased at a higher rate than did the entry of such drugs into the country.

So this is saying that the increase in supply caused the price movement, which is already mentioned in the premise. This does not advance the argument. (C) is out.
Quote:
(D) The wholesale price of a few illegal drugs increased substantially in 1987.

This is again similar to B. (D) is out.
Quote:
(E) A drop in demand for most illegal drugs in 1987 was not the sole cause of the drop in their wholesale price.

This matches our thought, removing the possible case that the eliminate the program as the cause of the price drop. Hence (E) is the correct answer.
Manager
Manager
Joined: 04 Mar 2020
Posts: 135
Own Kudos [?]: 129 [0]
Given Kudos: 304
Location: India
GMAT 1: 640 Q47 V30
Send PM
Re: The program to control the entry of illegal drugs into the country was [#permalink]
MBA2ran wrote:
The program to control the entry of illegal drugs into the country was a failure in 1987. If the program had been successful, the wholesale price of most illegal drugs would not have dropped substantially in 1987.

The argument in the passage depends on which of the following assumptions?


(E) A drop in demand for most illegal drugs in 1987 was not the sole cause of the drop in their wholesale price.



Hi VeritasKarishma

I chose 'E', however, I was a little skeptical about the verbiage. Don't you think the usage "...was not the solecause of the drop..." is extreme ? I mean, author doesn't have to assume that demand was not the "sole" cause of the drop. It could have been one of the 2 causes, Right ?
Tutor
Joined: 16 Oct 2010
Posts: 14817
Own Kudos [?]: 64899 [0]
Given Kudos: 426
Location: Pune, India
Send PM
Re: The program to control the entry of illegal drugs into the country was [#permalink]
Expert Reply
ShreyKapil08 wrote:
MBA2ran wrote:
The program to control the entry of illegal drugs into the country was a failure in 1987. If the program had been successful, the wholesale price of most illegal drugs would not have dropped substantially in 1987.

The argument in the passage depends on which of the following assumptions?


(E) A drop in demand for most illegal drugs in 1987 was not the sole cause of the drop in their wholesale price.



Hi VeritasKarishma

I chose 'E', however, I was a little skeptical about the verbiage. Don't you think the usage "...was not the solecause of the drop..." is extreme ? I mean, author doesn't have to assume that demand was not the "sole" cause of the drop. It could have been one of the 2 causes, Right ?


Option (E) makes sense because it uses the word "sole". I don't know why many test takers think in terms of "the option is extreme" but it is not valid. There are no extreme options. In some cases, words such as "all", "none" etc are justified and in others, they are not - just like any other word. You need to see whether it is justified in the case presented to you.

When the author says that price drop in 1987 implies that supplies continued at a healthy level or even increased, he is assuming that the reason for the price drop is related to the supply side. This means he is assuming that fall in demand is not the sole reason.
Director
Director
Joined: 23 Apr 2019
Status:PhD trained. Education research, management.
Posts: 806
Own Kudos [?]: 1807 [0]
Given Kudos: 203
Send PM
Re: The program to control the entry of illegal drugs into the country was [#permalink]
OFFICIAL GMAT EXPLANATION

The only choice that must be true in order to conclude legitimately from the drop in wholesale price of illegal drugs that the program was a failure is choice E, the best answer. If the drop in price was caused by a drop in demand, there is no reason to suspect that there has been any increase in supply caused by drugs entering the country.

The other choices can be false without affecting the argument. The supply of illegal drugs need not have dropped (choice A), and the retail price could have dropped (choice B). The entry of illegal drugs could have risen at a higher rate than domestic production (choice C), and no illegal drug need have undergone a substantial price rise (choice D).
Intern
Intern
Joined: 29 May 2020
Posts: 16
Own Kudos [?]: 7 [0]
Given Kudos: 94
Send PM
Re: The program to control the entry of illegal drugs into the country was [#permalink]
The program to control the entry of illegal drugs into the country was a failure in 1987. If the program had been successful, the wholesale price of most illegal drugs would not have dropped substantially in 1987.

The argument in the passage depends on which of the following assumptions?


(A) The supply of illegal drugs dropped substantially in 1987.

(B) The price paid for most illegal drugs by the average consumer did not drop substantially in 1987.

(C) Domestic production of illegal drugs increased at a higher rate than did the entry of such drugs into the country.

(D) The wholesale price of a few illegal drugs increased substantially in 1987.

(E) A drop in demand for most illegal drugs in 1987 was not the sole cause of the drop in their wholesale price.


The program to control the entry of illegal drugs into the country was a failure in 1987. If the program had been successful, the wholesale price of most illegal drugs would not have dropped substantially in 1987.


the program failed so illegal drugs increased in supply. By saying that the prices would not have dropped if the program had been successful, the argument assumes that increased supply was the reason that decreased the price and does not consider other reasons that could have decreased the price regardless whether the program had or had not been implemented.

usually when they talk about price, supply, and demand, you can prethink the answer.
there are two ways prices can decrease when considering supply and demand.
if supply increases, prices decrease
if demand decreases, prices decrease

clearly the argument did not consider the following option:
if demand decreases, prices decrease

(E) A drop in demand for most illegal drugs in 1987 was not the sole cause of the drop in their wholesale price.
Manager
Manager
Joined: 21 Jul 2020
Posts: 69
Own Kudos [?]: 12 [0]
Given Kudos: 3
WE:Operations (Manufacturing)
Send PM
Re: The program to control the entry of illegal drugs into the country was [#permalink]
The program to control the entry of illegal drugs into the country was a failure in 1987. If the program had been successful, the wholesale price of most illegal drugs would not have dropped substantially in 1987.

Conclusion: the wholesale price of most illegal drugs would not have dropped substantially in 1987. " If amount of drugs coming into the country increases, this means a supply increase in drugs, and will result in a lower price for it. supply vs demand.

The argument in the passage depends on which of the following assumptions?


(A) The supply of illegal drugs dropped substantially in 1987.wrong. supply should increase.

(B) The price paid for most illegal drugs by the average consumer did not drop substantially in 1987.the price paid is not an assumption that is needed.

(C) Domestic production of illegal drugs increased at a higher rate than did the entry of such drugs into the country.we are talking about drugs entering the country, not domestic production.

(D) The wholesale price of a few illegal drugs increased substantially in 1987.price should have decreased.

(E) A drop in demand for most illegal drugs in 1987 was not the sole cause of the drop in their wholesale price. if a drop in demand WAS the sole cause of the drop, the argument fails. This is the right answer.
Manager
Manager
Joined: 20 Feb 2019
Posts: 79
Own Kudos [?]: 32 [0]
Given Kudos: 182
Location: India
GMAT 1: 670 Q49 V33
GMAT 2: 710 Q49 V38
GPA: 3.2
Send PM
Re: The program to control the entry of illegal drugs into the country was [#permalink]
MBA2ran wrote:
The program to control the entry of illegal drugs into the country was a failure in 1987. If the program had been successful, the wholesale price of most illegal drugs would not have dropped substantially in 1987.

The argument in the passage depends on which of the following assumptions?


(A) The supply of illegal drugs dropped substantially in 1987.

(B) The price paid for most illegal drugs by the average consumer did not drop substantially in 1987.

(C) Domestic production of illegal drugs increased at a higher rate than did the entry of such drugs into the country.

(D) The wholesale price of a few illegal drugs increased substantially in 1987.

(E) A drop in demand for most illegal drugs in 1987 was not the sole cause of the drop in their wholesale price.

Related question (weaken): https://gmatclub.com/forum/the-program-to-control-the-entry-of-illegal-drugs-into-the-weaken-137754.html

The only choice that must be true in order to conclude legitimately from the drop in wholesale price of illegal drugs that the program was a failure is choice E, the best answer. If the drop in price was caused by a drop in demand, there is no reason to suspect that there has been any increase in supply caused by drugs entering the country.

The other choices can be false without affecting the argument. The supply of illegal drugs need not have dropped (choice A), and the retail price could have dropped (choice B). The entry of illegal drugs could have risen at a higher rate than domestic production (choice C), and no illegal drug need have undergone a substantial price rise (choice D).



Hello experts,

C'. Domestic production of illegal drugs DID NOT increase at a higher rate than did the entry of such drugs into the country.

Can this be an assumption too?

VeritasKarishma ScottTargetTestPrep AndrewN
Volunteer Expert
Joined: 16 May 2019
Posts: 3512
Own Kudos [?]: 6857 [0]
Given Kudos: 500
Re: The program to control the entry of illegal drugs into the country was [#permalink]
Expert Reply
ashmit99 wrote:
The argument in the passage depends on which of the following assumptions?
Hello experts,

C'. Domestic production of illegal drugs DID NOT increase at a higher rate than did the entry of such drugs into the country.

Can this be an assumption too?

VeritasKarishma ScottTargetTestPrep AndrewN

Hello, ashmit99. You have to read the question stem carefully to avoid associative reasoning that can lead you wayward. We are not interested in assumptions that could be made, but in the one assumption of the options provided that the argument depends on. In other words, if such an answer were untrue, then the argument as presented would collapse. I would hope that the earlier responses in the thread help clear any doubts about (C), but if you still have questions, let me know.

- Andrew
Manager
Manager
Joined: 20 Feb 2019
Posts: 79
Own Kudos [?]: 32 [0]
Given Kudos: 182
Location: India
GMAT 1: 670 Q49 V33
GMAT 2: 710 Q49 V38
GPA: 3.2
Send PM
Re: The program to control the entry of illegal drugs into the country was [#permalink]
AndrewN wrote:
ashmit99 wrote:
The argument in the passage depends on which of the following assumptions?
Hello experts,

C'. Domestic production of illegal drugs DID NOT increase at a higher rate than did the entry of such drugs into the country.

Can this be an assumption too?

VeritasKarishma ScottTargetTestPrep AndrewN

Hello, ashmit99. You have to read the question stem carefully to avoid associative reasoning that can lead you wayward. We are not interested in assumptions that could be made, but in the one assumption of the options provided that the argument depends on. In other words, if such an answer were untrue, then the argument as presented would collapse. I would hope that the earlier responses in the thread help clear any doubts about (C), but if you still have questions, let me know.

- Andrew


Thanks for replying AndrewN

Got your point. But if I negate C' it would become the original option C.

So C says that domestic production increased at higher rate than did entry of such drugs into the country.

So basically now it seems like even if the program had been successful, the wholesale price still would have dropped substantially.

So in this case, the conclusion breaks.

Please help me with my doubt.

Thanks!
Volunteer Expert
Joined: 16 May 2019
Posts: 3512
Own Kudos [?]: 6857 [3]
Given Kudos: 500
Re: The program to control the entry of illegal drugs into the country was [#permalink]
2
Kudos
1
Bookmarks
Expert Reply
ashmit99 wrote:
Thanks for replying AndrewN

Got your point. But if I negate C' it would become the original option C.

So C says that domestic production increased at higher rate than did entry of such drugs into the country.

So basically now it seems like even if the program had been successful, the wholesale price still would have dropped substantially.

So in this case, the conclusion breaks.

Please help me with my doubt.

Thanks!

Hello again, ashmit99. I did not say anything about negation exactly. Your goal is simply to find the one answer choice that necessarily bridges the gap between premise and conclusion. I use a drag-and-drop (or missing link) strategy instead of the negation technique when taking on such questions. Test (C) as is between the premise and conclusion:

Premise: If the program had been successful, the wholesale price of most illegal drugs would not have dropped substantially in 1987.

(C) Domestic production of illegal drugs increased at a higher rate than did the entry of such drugs into the country.

Conclusion: The program to control the entry of illegal drugs into the country was a failure in 1987.

Analysis: Why are we focused on a rate of growth between the domestic production of drugs and the importation of drugs? Whether the domestic production outpaced the importation is beside the point. The conclusion about the program to control the entry of illegal drugs into the country could hold, regardless of what may be going on with domestic production. Answer choice (C) cannot be our necessary assumption.

Repeat the process for (E):

Premise: If the program had been successful, the wholesale price of most illegal drugs would not have dropped substantially in 1987.

(E) A drop in demand for most illegal drugs in 1987 was not the sole cause of the drop in their wholesale price.

Conclusion: The program to control the entry of illegal drugs into the country was a failure in 1987.

Now this makes more sense. As evidence of the assertion, the argument points to the wholesale price of most illegal drugs. Choice (E) touches directly on this concern and informs us that demand for such drugs was not the sole cause of the drop in wholesale price, so there must have been other factors at work. Perhaps, then, the conclusion can be made that the program was a failure. At least the connection is clear, and that is all we are looking for in this sort of question.

I hope that makes more sense.

- Andrew
Tutor
Joined: 16 Oct 2010
Posts: 14817
Own Kudos [?]: 64899 [1]
Given Kudos: 426
Location: Pune, India
Send PM
Re: The program to control the entry of illegal drugs into the country was [#permalink]
1
Kudos
Expert Reply
ashmit99 wrote:
MBA2ran wrote:
The program to control the entry of illegal drugs into the country was a failure in 1987. If the program had been successful, the wholesale price of most illegal drugs would not have dropped substantially in 1987.

The argument in the passage depends on which of the following assumptions?


(A) The supply of illegal drugs dropped substantially in 1987.

(B) The price paid for most illegal drugs by the average consumer did not drop substantially in 1987.

(C) Domestic production of illegal drugs increased at a higher rate than did the entry of such drugs into the country.

(D) The wholesale price of a few illegal drugs increased substantially in 1987.

(E) A drop in demand for most illegal drugs in 1987 was not the sole cause of the drop in their wholesale price.

Related question (weaken): https://gmatclub.com/forum/the-program-to-control-the-entry-of-illegal-drugs-into-the-weaken-137754.html

The only choice that must be true in order to conclude legitimately from the drop in wholesale price of illegal drugs that the program was a failure is choice E, the best answer. If the drop in price was caused by a drop in demand, there is no reason to suspect that there has been any increase in supply caused by drugs entering the country.

The other choices can be false without affecting the argument. The supply of illegal drugs need not have dropped (choice A), and the retail price could have dropped (choice B). The entry of illegal drugs could have risen at a higher rate than domestic production (choice C), and no illegal drug need have undergone a substantial price rise (choice D).



Hello experts,

C'. Domestic production of illegal drugs DID NOT increase at a higher rate than did the entry of such drugs into the country.

Can this be an assumption too?

VeritasKarishma ScottTargetTestPrep AndrewN


An option would need to be worded very carefully to make it an assumption.

Other than option (E), an assumption is: "Dramatic increase in domestic production was not the sole cause of price drop."

Rate of increase in domestic production vs rate of increase in illegal import doesn't make much sense.
Intern
Intern
Joined: 22 Mar 2020
Posts: 27
Own Kudos [?]: 1 [0]
Given Kudos: 71
Send PM
Re: The program to control the entry of illegal drugs into the country was [#permalink]
AndrewN wrote:
ashmit99 wrote:
Thanks for replying AndrewN

Got your point. But if I negate C' it would become the original option C.

So C says that domestic production increased at higher rate than did entry of such drugs into the country.

So basically now it seems like even if the program had been successful, the wholesale price still would have dropped substantially.

So in this case, the conclusion breaks.

Please help me with my doubt.

Thanks!

Hello again, ashmit99. I did not say anything about negation exactly. Your goal is simply to find the one answer choice that necessarily bridges the gap between premise and conclusion. I use a drag-and-drop (or missing link) strategy instead of the negation technique when taking on such questions. Test (C) as is between the premise and conclusion:

Premise: If the program had been successful, the wholesale price of most illegal drugs would not have dropped substantially in 1987.

(C) Domestic production of illegal drugs increased at a higher rate than did the entry of such drugs into the country.

Conclusion: The program to control the entry of illegal drugs into the country was a failure in 1987.

Analysis: Why are we focused on a rate of growth between the domestic production of drugs and the importation of drugs? Whether the domestic production outpaced the importation is beside the point. The conclusion about the program to control the entry of illegal drugs into the country could hold, regardless of what may be going on with domestic production. Answer choice (C) cannot be our necessary assumption.

Repeat the process for (E):

Premise: If the program had been successful, the wholesale price of most illegal drugs would not have dropped substantially in 1987.

(E) A drop in demand for most illegal drugs in 1987 was not the sole cause of the drop in their wholesale price.

Conclusion: The program to control the entry of illegal drugs into the country was a failure in 1987.

Now this makes more sense. As evidence of the assertion, the argument points to the wholesale price of most illegal drugs. Choice (E) touches directly on this concern and informs us that demand for such drugs was not the sole cause of the drop in wholesale price, so there must have been other factors at work. Perhaps, then, the conclusion can be made that the program was a failure. At least the connection is clear, and that is all we are looking for in this sort of question.

I hope that makes more sense.

- Andrew


Hi AndrewN

I was confused between B and E and chose B. My reason was-
If the average customer refused to pay high price for illegal drugs, the wholesale price of the commodity would have to be dropped. Where did I faulter in my reasoning here.
GMAT Club Bot
Re: The program to control the entry of illegal drugs into the country was [#permalink]
 1   2   
Moderators:
GMAT Club Verbal Expert
6919 posts
GMAT Club Verbal Expert
238 posts
CR Forum Moderator
832 posts

Powered by phpBB © phpBB Group | Emoji artwork provided by EmojiOne